Skin Benign lesions & Non-melanoma skin cancer Flashcards
A 55-year-old man presents with a fungating mass of the right shoulder that has been enlarging for the past 12 years. A photograph is shown. Medical history is unremarkable. The patient reports that he spent his youth working outside. He has a history of smoking. Which of the following is the most significant risk factor for development of this lesion in this patient?
A) CDK4 mutation
B) Previous use of topical steroids
C) Smoking history
D) Sun exposure
The correct response is Option D.
The patient presents with a nodular ulcerated basal cell carcinoma to the right shoulder. Basal cell carcinoma is the most common malignancy in the United States due to the increase in sun exposure and tanning salons. The most common risk factor for basal cell carcinomas is sun exposure. Generally, they do not metastasize and are resectable, but do lead to large oncologic resections if left to progress, as is the case in the patient described. Smoking history, although important, does not have a major impact on the risk for basal cell cancer. Marjolin ulcers are a variant of squamous cell carcinoma that results from the chronic inflammatory process that follows burns and is not a risk factor for basal cell carcinoma. Immunosuppression is a risk factor for basal cell carcinomas, but it is not as great as sun exposure and this patient has no history of immunosuppression. Other risk factors for basal cell carcinoma include being male and of older age, but these are not listed as possible choices. A mutation in the CDK4 gene is linked to familial melanoma and would have no relation to the basal cell tumor in this patient. Topical steroid use has no known impact on risk for development of basal cell carcinoma.
A 55-year-old man has a lesion on the right forearm that has enlarged over the past six weeks. A photograph is shown above. Which of the following is the most likely diagnosis?
(A) Cylindroma
(B) Dermatofibroma
(C) Keratoacanthoma
(D) Seborrheic keratosis
(E) Syringoma
The correct response is Option C.
The most likely diagnosis is keratoacanthoma, a common cutaneous neoplasm that typically occurs in men older than age 50 years. Keratoacanthomas grow rapidly over several weeks and are believed to regress spontaneously if left untreated. They are characterized by an umbilicated center with a keratin plug. Because they are difficult to distinguish from squamous cell carcinoma and may indeed be linked, excision for histopathologic confirmation is recommended.
Cylindromas are round, firm, fleshy tumors of the scalp that are rarely solitary.
Dermatofibroma is a fibrous, papular lesion characteristically found on the lower extremities in young adults.
Seborrheic keratosis has a waxy, greasy, or pressed-on appearance and generally occurs on the face and trunk of older persons.
Syringoma is a flesh-colored or yellow papule that typically develops in females during adolescence or early adulthood. These lesions may be multiple and often occur only on the lower eyelids.
A 75-year-old woman is evaluated because of a new skin lesion on the right upper eyelid. Examination of the specimen obtained on biopsy shows a 1-cm Merkel cell carcinoma. In addition to regional node sampling, which of the following is the most appropriate excision and adjuvant management in this patient?
A) 1-cm margins and chemotherapy
B) 2-cm margins and chemotherapy
C) 1-cm margins and postoperative radiation therapy
D) 2-cm margins and postoperative radiation therapy
E) 5-mm margins and postoperative radiation therapy
The correct response is Option C.
Merkel cell carcinoma, an aggressive neuroendocrine tumor, is most likely. It presents in older, immunocompromised women in sun-exposed areas. About 80% of Merkel cell carcinomas are secondary to polyomavirus infection. Treatment of the primary tumor should be wide local excision or Mohs micrographic surgery. For wide local excision of tumors smaller than 2 cm, the recommended surgical margin should be 1 cm. As there is a high rate of occult nodal metastasis, and nodal status is associated with mortality rates, biopsy of the sentinel node is recommended for all cases regardless of primary tumor size. Merkel cell carcinoma is a radiosensitive tumor, and postoperative adjuvant radiation therapy has been shown to decrease local recurrence. Chemotherapy is only currently indicated for palliation and distant metastasis.
A 60-year-old woman comes to the office for evaluation of a firm, violaceous, 3-cm lesion of the left cheek. An incisional biopsy shows a Merkel cell tumor. The residual margins are positive. Which of the following is the most appropriate management?
A) Chemotherapy
B) Laser ablation
C) Mohs micrographic surgery
D) Radiation therapy and chemotherapy
E) Wide local excision and radiation therapy
The correct response is Option E.
Merkel cell tumor is an unusual and highly aggressive skin cancer. More than 50% of Merkel cell tumors occur in the head and neck region. Risk factors for Merkel cell tumors are exposure to sun and ultraviolet light, and immunosuppression. There is a recent association with Merkel cell polyomavirus.
Merkel cell generally presents as a firm, painless nodule (up to 2 cm in diameter) or as a mass (>2 cm in diameter). Although classically described as red in color, it may be flesh-colored or blue. It often enlarges rapidly.
The standard of management is surgical excision combined with radiation therapy. Radiation therapy decreases local recurrence rates. Node-negative patients with no distant metastasis treated with surgery and radiation have 5-year survival rates of approximately 90%. Mohs micrographic surgery and wide local excision are both accepted modalities of surgical resection. It is well known that surgery alone is insufficient to cure or control Merkel cell tumors. Consideration should be given to evaluation of the lymph nodes. Sentinel node biopsy is a common modality.
A 67-year-old man comes to the office with biopsy-proven Merkel cell carcinoma of the forehead. In addition to wide resection, which of the following is the optimal treatment?
A) Administration of interferon
B) Injection of 5-fluorouracil
C) Neoadjuvant chemotherapy
D) Radiation therapy
The correct response is Option D.
Merkel cell carcinoma is a rare tumor that usually consists of smooth, painless, indurated, solitary dermal nodules approximately 2 to 4 mm in size. It occurs more frequently in patients older than age 65 years. Merkel cell carcinoma appears most often at sun-exposed sites on white skin; 50% occur on the head and neck, and 40% on the trunk. Merkel cell carcinoma is an aggressive tumor; metastases to regional lymph nodes are noted on initial diagnosis in 12 to 15% of patients. Regional metastasis eventually occurs in one half to two thirds of patients. Local recurrence following primary excision develops in 24 to 44% of patients. Time from diagnosis of the primary tumor to clinically apparent regional nodal metastases is approximately 7 to 8 months. Distant metastases occur ultimately in one third of patients; in order of frequency, metastases occur in the lymph, liver, bone, brain, lung, and skin. The mean time from diagnosis to systemic involvement is 18 months, with death occurring 6 months later. The 5-year survival rate has been reported as 30 to 64%. Two thirds or more of patients with local or regionally recurrent disease ultimately die. Surgical excision is the treatment of choice for primary tumors. The prevailing opinion regarding Merkel cell cancers is that they should be excised with margins similar to those for melanoma.
Sentinel lymph node biopsy is used in clinically node-negative patients with Merkel cell carcinoma.
Radiation should be considered for all patients with Merkel cell carcinoma, as it is radiosensitive. Injection of 5-fluorouracil and administration of interferon have never been shown to be effective in Merkel cell treatment.
In Merkel cell carcinoma, tumor width dictates surgical excision, not tumor depth of invasion (as in melanoma).
Mohs micrographic surgery has been advocated for resection of Merkel cell carcinoma, not only to obtain clear margins, but also to preserve the most amount of tissue.
A 75-year-old man comes to the office because of a 5-year history of a pruritic lesion on the right groin that has been enlarging gradually in size. A photograph is shown. Examination of a specimen obtained on biopsy shows Paget disease. Which of the following is the most appropriate next step in management?
A) Oral miltefosine
B) Topical hydrocortisone
C) Topical miconazole
D) Wide excision
E) Observation
The correct response is Option D.
This patient described has extramammary Paget disease and the treatment is wide excision. Paget disease of the breast also presents with eczematous skin changes and is associated with breast cancer. Extramammary Paget disease, however, is an intraepithelial carcinoma that commonly involves the vulvar, perianal, perineal, scrotal, and penile regions. It presents as well-defined, moist, erythematous plaques associated with pruritis. Histopathologic examination shows epidermal acanthosis and elongated rete ridges. Paget cells are large intraepidermal cells with a large nucleus and abundant pale cytoplasm. There is a 7 to 40% rate of associated malignancy. Wide excision is the standard of care, and recent reports have shown that Mohs micrographic surgery can improve evaluation of resection margins.
Observation is not appropriate for extramammary Paget disease given the potential risk for malignancy. Topical treatment with steroids or antifungals is also not appropriate. Oral miltefosine is a treatment for leishmaniasis, an infectious disease that can involve the skin, mucous membranes, and internal organs. Although cutaneous leishmaniasis can present with ulcerating lesions or a dense dermal infiltrate, the histology is predominantly histiocytes, lymphocytes, and plasma cells.
In a patient who has a halo nevus, which of the following is the primary indication for surgical excision?
(A) Elimination of circulating antibodies
(B) Premalignant potential
(C) Prevention of leukoderma
(D) Relief of pain
(E) Resemblance to melanoma
The correct response is Option E.
Halo nevi, so called because of the distinct “halo” area of depigmentation surrounding the benign nevus, are typically seen on the trunk in teenagers and young adults. Histologic examination will show nevus cells surrounded by a band-like infiltrate of lymphocytes that may completely obliterate the lesion. Halo nevi have no known premalignant potential; however, because melanomas can also develop an irregular, incomplete halo, excisional biopsy should be performed for any halo nevus that becomes enlarged, asymmetric, or ulcerated or displays other characteristics similar to melanoma. The depigmentation around the nevus, known as leukoderma, is typically not painful. Although circulating antibodies to melanoma have been found in patients with halo nevi, this discovery alone is not an indication for nevus excision.
A 35-year-old African American woman presents with multiple draining sinus tracts and nodular abscesses in the bilateral inframammary folds, groins, and axillae. She has failed topical and oral antibiotic therapies. Medical history includes type 2 diabetes mellitus, obesity, and keloid scarring. Which of the following is the most appropriate initial treatment for this patient?
A) Adalimumab therapy
B) Incision and drainage
C) Radiotherapy
D) Skin-tissue-saving excision with electrosurgical peeling (STEEP) procedure
E) Wide excision and skin grafting
The correct response is Option A.
This patient has hidradenitis suppurativa characterized by multiple nodules, abscesses, tunnels, and scars most commonly in the axillae, inframammary folds, groin, perigenital, and perineal region. Significant advances in medical therapy have decreased the need for surgical intervention. The disease is classified according to the Hurley classification: stage I as transient nonscarring inflammatory lesions; stage II as separate lesions consisting of recurrent abscesses with tunnel formation and scarring, and single or multiple lesions separated by normal-looking skin; and stage III as coalescent lesions with tunnel formation, scarring and inflammation. This patient is stage II. Mild disease is often treated with topical antibiotics (e.g., clindamycin) and/or resorcinol, while moderate disease may benefit from oral antibiotics (e.g., tetracycline 500 mg twice daily). Refractory disease may benefit from antibody therapy and/or surgical intervention.
Adalimumab (Humira) is a recombinant human igG1 anti-TNF monoclonal antibody that binds the proinflammatory cytokine TNF-alpha. It was approved by the FDA in 2016 for the treatment of moderate to severe disease where patients have required long-term antibiotics or rapid flares upon cessation of antibiotics. Alternative agents include infliximab (Remicade), anakinra (Kineret), ustekinumab (Stelara), dapsone, or acitretin (Soriatane).
Radiotherapy is incorrect. There are no randomized trials comparing radiotherapy with medical or surgical therapy, but it can be effective and well-tolerated for focal areas such as the scalp. The risk for secondary cancer is minimal but not negligible; therefore, radiotherapy is not considered a front-line option.
Surgical incision and drainage is indicated for fluctuant abscesses but not effective for inflamed nodules since there is no collection of fluid to drain.
Skin-tissue-saving excision with electrosurgical peeling (STEEP) or deroofing removes unhealthy tissue with step-wise tangential excisions and preserving normal tissue to heal via secondary intention. There is significant postoperative morbidity and risk for scarring with higher recurrence rates. Given this patient’s history for keloid scarring, this is not the best option.
Wide excision is associated with lower recurrence rates but higher morbidity (e.g., infection, bleeding, contractures). This can be effective in patients with areas of limited disease but should be reserved for patients with severe disease refractory to nonsurgical therapies when large total body surface area is involved. Nonsurgical options should be fully explored in this patient given the keloid history.
An otherwise healthy 20-year-old woman develops a keloid on her right earlobe after an ear piercing. Excision and radiation therapy are planned. Which of the following is the ideal time after excision for the initiation of radiation therapy?
A) 1 day
B) 7 days
C) 2 weeks
D) 4 weeks
E) 6 weeks
The correct response is Option A.
The ideal time to give radiation therapy in this case is on postoperative day one. Earlobes and the helix of the auricle are common sites for keloid formation, usually after trauma or ear piercing, with an incidence of approximately 2.5%. There are numerous adjuvant therapies (eg, radiation therapy), medical therapies, (eg, intralesional steroids, 5-fluorouracil interferon, and topical silicone), and physical approaches (eg, pressure) that can be used in addition to excision of the keloid to help reduce its recurrence. However, the treatment options for such lesions are still controversial, because there are numerous challenges, and no single best treatment or best combination of treatments has been proved to manage these conditions effectively.
Radiation therapy has long been known to be effective in the early phase of wound healing. More specifically, radiation therapy is sensitive to endothelial vascular buds and decreases proliferation of new fibroblasts. As such, radiation therapy after keloid excision should be performed as soon as possible. Usually, better results are reported when radiation therapy is performed within 1 to 3 days after surgery. The most commonly administered doses are between 10 and 15 Gy over a period of 2 or 3 days.
The other options are incorrect as they are outside the ideal time frame.
A 63-year-old woman with a history of ulcerative colitis presents after being diagnosed with invasive ductal carcinoma of the right breast. Her BMI is 23 kg/m2. She undergoes unilateral mastectomy with immediate reconstruction with a free deep inferior epigastric perforator flap. Her initial postoperative course is uneventful. Five weeks after the procedure, the patient returns to the clinic. Examination shows erythema and skin ulcers surrounded by violaceous discoloration around the abdominal and breast incisions. No underlying fluid collections are appreciated. Punch biopsies of the ulcers are performed and show neutrophilic dermatosis. Which of the following is the most appropriate next step in management?
A) Broad-spectrum antibiotic therapy
B) CT scan of the chest/abdomen/pelvis
C) High-dose steroid therapy
D) Surgical exploration with washout and debridement
E) Ultrasound of the breast and abdomen
The correct response is Option C.
Pyoderma gangrenosum (PG) is a rare skin disorder of unknown cause that is believed to be part of the spectrum of neutrophilic dermatoses. Specific characteristics of postoperative PG include dramatic deterioration of surgical wounds after a period of 4 days to 6 weeks, with predominant involvement of the breast and abdomen. Numerous reports of PG following reduction mammaplasty and breast reconstruction have been published. Additionally, patients with a history of inflammatory bowel disease are at an increased risk for developing PG.
Clinically, postoperative PG is characterized by fever, chills, cellulitis, and wound breakdown. The hallmark finding is a rapidly enlarging area of central skin ulceration surrounded by violaceous skin with irregular borders. Systemic signs of inflammation along with laboratory abnormalities (leukocytosis, hyponatremia, and hypoproteinemia) can accompany these findings. The diagnostic challenge of postoperative PG is related to its clinical resemblance to necrotizing infection and, hence, a high index of suspicion is required. However, the presence of neutrophilic dermatosis supports the diagnosis of PG. Treatment consists of immunosuppression and frequently consists of a combination of high-dose steroids and topical tacrolimus.
Additional imaging would not aid in management in this case. Broadening antibiotic coverage or surgical debridement would be indicated for treatment of infection. Local wound care does not address the underlying cause of the clinical condition and, hence, would not be the most appropriate next step.
A 75-year-old man presents with an abnormal skin lesion. Biopsy confirms Merkel cell carcinoma. Which of the following best describes the characteristics of this skin cancer?
A) The behavior is aggressive with high rates of lymph node spread and local recurrence
B) The plantar surface of the foot is the most common location for this lesion
C) This disease is more common amongst patients of African descent
D) This lesion frequently arises within the setting of a chronic wound
E) Topical chemotherapeutic agents are the primary method for managing this condition
The correct response is Option A.
Merkel cell carcinoma (MCC) is an uncommon skin cancer which has aggressive behavior. MCC usually presents as a painless red nodule that is rapidly growing. Elderly patients are more commonly affected by MCC, and it is believed that sun exposure is a major risk factor. There are several studies that suggest involvement of the polyomavirus in the pathogenesis of MCC. Patients with immunosuppression are also at relatively higher risk for MCC. This is an aggressive skin cancer with early and frequent involvement of the regional lymph nodes. Patients with large tumors or regional spread are at high risk for distant metastasis. This disease is significantly more common in lighter-skinned patients than in dark-skinned patients. The head and neck region is the most common site for MCC. A Marjolin’s ulcer is a rare and aggressive type of squamous cell carcinoma arising in the setting of a chronic wound. Once a diagnosis has been established, treatment of MCC typically involves wide local excision and lymph node mapping. Many of the treatment strategies for MCC are similar to melanoma given the similar aggressive clinical behavior.
A neonate has a 4 * 4-cm congenital defect of the scalp and underlying skull. The brain is visible beneath a gray membrane. Which of the following is the most appropriate initial management?
(A) Frequent application of silver sulfadiazine ointment
(B) Wet-to-dry dressing changes twice daily
(C) Biopsy of the wound margins
(D) Tissue expansion and coverage with scalp flaps
The correct response is Option A.
This neonate has cutis aplasia, or congenital absence of the layers of the skin and scalp that can also expand to include the skull. Conservative management is most appropriate; frequent application of silver sulfadiazine ointment and coverage with occlusive dressings will allow for wound healing. Some surgeons also advocate skin grafting over the exposed areas to prevent wound desiccation.
Use of dry dressings can actually result in desiccation of the dura and rupture of the sagittal sinus, a fatal complication. In patients with cutis aplasia, the wound must be kept moist at all times. Biopsy of the wound margins is not indicated because this patient does not have a malignant or premalignant condition. Tissue expansion is not required for this small wound, which will heal by secondary intention without reconstruction.
A 2-year-old boy is brought to the emergency department because of a 2-day history of lethargy, fever, and a reddish purple rash on his arms and legs. Temperature is 103.0°F (39.4°C). Physical examination shows a petechial rash on the upper and lower extremities and trunk. Broad-spectrum antibiotics are initiated. After 6 hours, the rash begins to hemorrhage and blister and his digits become ischemic. Which of the following is the most appropriate management?
A ) Activated protein C
B ) Amputation of the ischemic digits
C ) Avoidance of inotropic support
D ) Debridement of skin
E ) Thrombolytics
The correct response is Option A.
Purpura fulminans is a rapidly evolving autoimmune syndrome of septic shock and hemorrhagic bullae that can result in massive desquamation and is frequently fatal. Management includes prompt recognition of the infection (which is usually due to Neisseria meningitidis), initiation of broad-spectrum antibiotics, mechanical ventilation, and aggressive fluid resuscitation with inotropic support. Patients develop disseminated intravascular coagulopathy and appear to benefit from replacement of activated protein C. A recently published, multicenter retrospective review of 70 patients documented an amputation rate of 90% and suggested the need for early fasciotomy to improve limb salvage. It is very difficult to determine tissue viability during the resuscitation period; therefore, amputation, debridement, and coverage are delayed until demarcation has occurred. Thrombolytics are not used in this situation because the patient has a hemorrhagic disorder.
A 14-year-old girl with Fitzpatrick Type V skin comes to the office for evaluation of a nevus on the right side of the face. Physical examination shows a macular, bluish grey, irregular area of hyperpigmentation involving the right infrapalpebral region, nasolabial fold, and zygomatic region. Pigmentation of the right sclera is noted. Which of the following is the most appropriate treatment for this lesion?
A) Camouflage therapy
B) Cryotherapy
C) Dermabrasion
D) Mohs micrographic excision
E) Q-switched ruby laser
The correct response is Option E.
This patient has the acquired form of nevus of Ota, also known as nevus fuscoceruleus ophthalmomaxillaris or oculodermal melanocytosis, a dermal melanocytic hamartoma that demonstrates bluish hyperpigmentation along the ophthalmic and maxillary divisions of the trigeminal nerve. The failure of complete embryonic migration of melanocytes from the neural crest to the epidermis results in dermal nesting with the resultant dermal melanin causing the Tyndall effect. This disorder primarily affects darker-pigmented individuals and is more prevalent in females. It has a bimodal age incidence, with a peak at 1 year of age and a second around puberty. The lesion tends to become increasingly prominent with age, puberty, and postmenopausal state. Most patients have no family history. Ophthalmologic examination is recommended because of a reported 10% association with ipsilateral glaucoma. Malignant degeneration to melanoma occurs in approximately 4% of reported cases and is more frequent in lighter-skinned individuals. Diagnosis is mainly clinical with confirmatory biopsy indicated when the diagnosis is uncertain or in rapidly expanding or nodular lesions suggestive of malignancy.
The most effective treatment option is laser therapy, particularly with a Q-switched laser with ruby (694 nm), alexandrite (755 nm), or neodymium: yttrium-aluminum-garnet (1064 nm). The wavelength, pulse duration, and energy densities inherent in the Q-switched laser provide the desired parameters for melanin photothermolysis. Dyspigmentation is a possible complication, although it is mostly transient.
Before the advent of laser therapy, treatment options were suboptimal. Makeup or camouflage therapy offered only temporary improvement. Dermabrasion followed by cryotherapy had the potential for dermal scarring and atrophy and was ineffective for those lesions with deep dermal melanocytes. Surgical excision options were also associated with scarring. Mohs micrographic excision has not been described for excision of these lesions.
A 72-year-old man is being evaluated because of a 3-month history of lesions on the nasal dorsum and cheek (shown). He is a poor surgical candidate and is treated with imiquimod (Aldara). Which of the following is the most likely mechanism of action of this treatment?
A ) Inhibition of the cyclooxygenase pathway
B ) Inhibition of DNA synthesis
C ) Modification of gene transcription
D ) Modulation of cell differentiation
E ) Stimulation of proinflammatory cytokine production
The correct response is Option E.
Imiquimod (Aldara) is a new immune response enhancer that stimulates host cytokine production and induces apoptosis of tumor cells. It has been used to treat actinic keratoses, viral warts, and nonmelanoma skin malignancy. Topical 5-fluorouracil is a topical chemotherapeutic agent that directly inhibits DNA synthesis. Retinoids prevent new skin cancer development by regulating cell differentiation. Topical diclofenac is an anti-inflammatory drug that inhibits the cyclooxygenase pathway and has been found useful in the treatment of actinic keratoses. Interferons control cell differentiation by modification of gene transcription and have been used in combination with retinoids for advanced squamous cell cancers.
Which of the following subtypes of basal cell carcinoma has the lowest risk for local recurrence?
A) Infiltrating
B) Micronodular
C) Morpheaform
D) Nodular
E) Sclerosing
The correct response is Option D.
Basal cell carcinoma (BCC) is the most common human malignancy as well as the most common malignant tumor of the skin. The incidence of BCC is increasing worldwide. The most significant etiological factors are believed to be ultraviolet light exposure and genetic predisposition. Therefore, an aging population and prolonged exposure to sunlight may explain the worldwide increasing incidence.
The great majority of BCCs are successfully treated and will not recur. However, it is important to know the high risk lesions and subtypes to understand when the recurrence rate may be higher. The morpheaform, sclerosing, infiltrating, micronodular, and metatypical subtypes are associated with higher risk for relapse. Anatomic locations on the trunk and limbs are considered low-risk areas, while the forehead, cheek, chin, scalp, and neck are intermediate-risk areas. The nose and perioral areas are high-risk areas. Size greater than 1 cm for head and neck tumors and greater than 2 cm in other body areas also predisposes to a higher recurrence risk.
A 38-year-old woman has severe hidradenitis suppurativa of the groin and axillae. Which of the following dermal appendages are located in these areas and implicated in the disease process?
A ) Apocrine glands
B ) Eccrine glands
C ) Glomus bodies
D ) Hair follicles
E ) Sebaceous glands
The correct response is Option A.
Apocrine glands are uniquely located in the axillae, groin, and perineum, and they secrete a viscid, milky fluid that becomes malodorous with bacterial colonization. Occlusion of the glands causes inflammation and subcutaneous abscess formation with pain, drainage, and foul odor. This crippling disease can be medically managed with chronic suppressive topical and systemic antibiotics but often requires intermittent incision and drainage, or even surgical resection.
Eccrine glands are found throughout the skin, secreting a thin, clear, hypotonic fluid ( €œsweat €). Glomus bodies are located in tissues exposed to the cold, such as the fingertips and ears. They form a thickening in the arterial wall before naturally occurring thermoregulatory arteriovenous shunts, and they are thought to control the flow through these shunts. Although some recent data suggest that hair follicle occlusion and folliculitis lead to apocrine gland occlusion and subsequent hidradenitis suppurativa, hair follicles are located all over the body and do not form the abscesses responsible for the clinical disease. Sebaceous glands secrete sebum, an oily substance that lubricates hair follicles and surrounding skin. They are found throughout the skin, except for the palms and soles. They are found in abundance in the face and scalp.
A 40-year-old man comes to the office because of a 12-year history of recurrent painful nodules in his groin, buttocks, and perineum. Physical examination shows deep subcutaneous abscesses. Some have ruptured and formed multiple discharging sinus tracts. In addition to meticulous hygiene, which of the following is the most appropriate management?
A) Antiandrogen therapy
B) Anti-tumor necrosis factor-a therapy
C) Excision
D) Radiotherapy
E) Regular chlorhexidine baths
The correct response is Option C.
The patient has hidradenitis suppurativa. The disease presents with tender subcutaneous nodules beginning around puberty. The nodules may spontaneously rupture or coalesce, forming deep, painful dermal abscesses. Eventually, fibrosis and the formation of extensive sinus tracts result. The location of the lesions may lead to social embarrassment.
Due to the multiple interconnected sinus tracts and abscesses throughout an entire region, the patient described has such a debilitating disease that only surgery can adequately address his symptoms. Wide excision of all affected tissue and the underlying sinus tracts is the most effective treatment. It is also advisable to stage the process, preferably with the use of allograft.
For patients with abscesses but no cicatrization or sinuses, hygienic measures and antibiotics are an appropriate first-line therapy. The disease primarily involves the follicular epithelium, which is colonized secondarily and infected by bacteria. Clindamycin and tetracycline have shown benefit in clinical trials. Smaller studies using dapsone and minocycline have shown useful short-term benefit.
Several trials using antiandrogen therapy have been conducted. In a randomized trial comparing ethinyl estradiol 50 mg/cyproterone acetate 50 mg to ethinyl estradiol 50 mg/norgestrel 500 mg in 24 women, both regimens produced improvement in disease activity. The tumor necrosis factor-a inhibitors infliximab and etanercept have also produced favorable outcomes.
Limited lesions can be injected with corticosteroids, and flares can be addressed with short courses of oral or intramuscular corticosteroids. Patients with one or more widely separated, recurrent abscesses with sinus tract formation and scars or patients who have failed the first-line therapies may need more aggressive treatment than those with early stage lesions. Treatments that carry more risk may be worth trying depending on the severity of the patient’s disease course. Long-term immunosuppressive therapy or surgical therapies, such as limited excisions or the laying open of sinus tracts, may be helpful.
Radiation therapy for hidradenitis suppurativa was used extensively in the past. Techniques and responses have varied widely, but poor tissue healing was noted.
A 13-year-old girl is brought for evaluation because of a 4-month history of severe pain of the tip of the right index finger. There is no history of trauma. On examination, the finger appears normal with no visible swelling or discoloration. The pain is exacerbated by local pressure when the patient writes and during her weekly swimming lessons. MRI (T2-weighted) image is shown. Which of the following is the most likely diagnosis?
A) Digital fibroma
B) Giant cell tumor
C) Glomus tumor
D) Neuroma
E) Venous malformation
The correct response is Option C.
This lesion is a glomus tumor. Glomus tumors are benign hamartomas originating from the glomus body, a structure comprised of vascular and neural elements that is responsible for thermoregulation in the skin. These often inconspicuous tumors present with pain, point tenderness, and sensitivity to cold. X-ray studies may show cortical erosion of the bone adjacent to the lesion, and ultrasonography can provide confirmation. MRI is the most accurate imaging modality and the tumor appears as a bright, discrete mass on T2-weighted images.
Digital fibroma is a cutaneous fibroblastic proliferation and would be visible. It rarely causes pain and would not enhance on T2-weighted MRI imaging. Neuroma can cause focal pain as described in the vignette, but the enhancing focal lesion on the T2-weighted MRI effectively rules out solid masses such as neuroma or giant cell tumor (which is common but rarely causes pain or cold intolerance). Venous malformation would enhance on T2-weighted MRI, but would typically present with swelling and would not be as well circumscribed as the lesion shown here.
A 45-year-old woman comes to the office for consultation regarding multiple actinic keratoses on the face. Physical examination shows fair-skinned complexion and small scaly patches on the face and ears. Which of the following interventions will result in the most desirable long-term aesthetic appearance in this patient?
(A) Cryosurgery
(B) Electrodesiccation and curettage
(C) Excision of the lesions with 2-mm margins
(D) Microdermabrasion
(E) Topical application of 0.5% fluorouracil cream
The correct response is Option E.
Several formulations and concentrations of topical fluorouracil have received U.S. Food & Drug Administration (FDA) approval for the treatment of keratotic lesions. Fluorouracil 5% and 0.5% creams have demonstrated, in clinical trials, a marked ability to eradicate keratotic lesions. The cosmetic result is often far better than surgical excision. Irritation at the application site, erythema, and burning are common side effects of both formulations, but comparative data suggest that the fluorouracil 0.5% cream is more cost €‘effective and may be safer, more tolerable, and as effective as fluorouracil 5% cream. Actinic keratosis is a common sun €‘induced skin disease. Recent molecular studies indicate an association between actinic keratosis and squamous cell carcinoma. Although 60% of squamous cell carcinoma cases begin as actinic keratosis, the risk of progression to squamous cell carcinoma is minimal. Other methods of treatment such as cryosurgery, topical trichloracetic acid, and curettage should be used with caution because hypopigmentation with scarring may result. Microdermabrasion is not an acceptable treatment for actinic keratosis.
A 47-year-old man comes to the office because of an asymptomatic lesion of the anterior abdominal wall that has been enlarging gradually for the past 10 years. The lesion has accelerated in growth during the past several months and recently ulcerated. Examination shows a 6-cm, raised, indurated, and irregularly shaped violaceous plaque consisting of firm, irregular nodules. Examination of a specimen obtained on incisional biopsy shows a soft-tissue malignancy arising from mesenchymal cells in the dermis. Which of the following is the most appropriate treatment?
A) Wide local incision and molecular targeted therapy
B) Wide local excision and sentinel node biopsy
C) Wide local excision, molecular targeted therapy, and radiation therapy
D) Wide local excision only
E) Wide local excision, sentinel node biopsy, and chemotherapy
The correct response is Option D.
The diagnosis of the described lesion is dermatofibrosarcoma protuberans (DFSP). It accounts for less than 0.1% of all malignant neoplasms and approximately 1% of all soft-tissue sarcomas, but is the most common type of cutaneous sarcoma. It is a malignant mesenchymal tumor that arises in the dermis and is characterized by latency in its initial detection, slow infiltrative growth, and local recurrence if not adequately treated. Distant metastasis is rare and generally occurs as a late sequela after repeated local recurrences. DFSP is most commonly found on the trunk followed by the proximal extremities, and rarely in the head and neck. These tumors have irregular shapes, frequent finger-like extensions, and an infiltrating growth pattern extending beyond clinical margins that result in incomplete removal and a propensity for local recurrence. Treatment primarily consists of wide surgical excision to include margins of 2 to 3 cm beyond the clinical tumor border if possible. Mohs micrographic surgery has been used with good outcomes in aesthetically sensitive areas such as the head and neck where tissue sparing is important. Reconstruction with tissue rearrangement or flaps should be performed after negative margins are confirmed. Most recurrences occur within 3 years of the primary excision, and close follow-up is indicated.
Conventional chemotherapy is rarely used. Radiation therapy is used as an adjunct to surgery for close or positive margins in areas where adequate wide resection alone may result in major cosmetic or functional deficits. Molecular targeted therapy such as imatinib mesylate (Gleevec) is indicated for unresectable, recurrent, or metastatic DFSP. Sentinel node biopsy is not indicated in the treatment for DFSP.
Keratoacanthoma is a subtype of which of the following tumors?
(A) Basal cell carcinoma
(B) Malignant melanoma
(C) Merkel cell tumor
(D) Sebaceous carcinoma
(E) Squamous cell carcinoma
The correct response is Option E.
Keratoacanthoma is a subtype of squamous cell carcinoma. It is well differentiated in most cases, but can exhibit a rapid growth phase with extensive tissue destruction. This is particularly problematic in the central area of the face, where aggressive perineural, subcutaneous, and intravenous invasion occurs. A rapid growth phase may occur over a period of a few weeks and can be followed by a dormant period. Complete spontaneous resolution can occur in the area without recurrence, although this resolution can leave a sizable area of destruction. These lesions are usually solitary and occur in sun-exposed areas in both men and women older than age 40 years. The hallmark physical feature of keratoacanthoma is a central destructive crater with a distinct fleshy rim. A central plug of keratin can also be associated.
Sebaceous carcinomas are uncommon skin malignancies that arise most frequently on the eyelids. These tumors contain sebaceous cells and are typically organized into circumscribed lobules. Periorbital primary lesions can act aggressively with high rates of early metastasis. Merkel cell tumor is a neuroendocrine carcinoma that is biologically aggressive and usually involves the head and neck area. Although basal cell carcinoma and malignant melanoma are not generally associated with keratoacanthoma, they can be seen in combination with keratoacanthoma because all of these tumors arise in sun-exposed areas.
A 56-year-old woman presents with a 1-cm primary superficial basal cell carcinoma on the left volar mid-forearm. Medical history includes renal transplantation, carcinoma of the right breast managed with lumpectomy and radiation, and treatment for a gunshot wound to the left forearm. The basal cell carcinoma is located within the previous traumatic scar. Which of the following clinical features is an indication for Mohs micrographic surgery in this patient?
A) Basal cell carcinoma arising in traumatic scar
B) History of radiation
C) Immunocompromised status of patient
D) Size of basal cell carcinoma
E) Superficial basal cell carcinoma
The correct response is Option A.
The clinical feature in this particular patient that fulfills the criteria to get Mohs micrographic surgery is that the basal cell carcinoma is arising in a traumatic scar. There are many clear indications for Mohs micrographic surgery for basal cell carcinoma: certain size, histology, and anatomic location, all recurrent basal cell carcinomas, and the occurrence of basal cell carcinoma in irradiated skin, traumatic scars, areas with osteomyelitis/chronic ulceration/inflammation, and/or patients with genetic syndromes. Almost all primary basal cell carcinomas in the H and M zones, regardless of pathology (i.e. superficial, nodular, or aggressive), size, or health status of the patient, are candidates for Mohs micrographic surgery. In the L zone, most basal cell carcinomas are also candidates for Mohs micrographic surgery (except all superficial subtypes [irrespective of health of patient], or those less than 1 cm size in immunocompromised patients or nodular subtypes).
Area H: “Mask areas” of face (central face, eyelids [including inner/outer canthi], eyebrows, nose, lips [cutaneous/mucosal/vermilion], chin, ear and periauricular skin /sulci, temple), genitalia (including perineal and perianal), hands, feet, nail units, ankles, and nipples/areola.
Area M: Cheeks, forehead, scalp, neck, jawline, and pretibial surface.
Area L: Trunk and extremities (excluding pretibial surface, hands, feet, nail units and ankles).
In this clinical case, the patient has a basal cell carcinoma that has a favorable pathology (i.e. superficial subtype) in the L zone, not an indication for Mohs micrographic surgery, irrespective of tumor size. Additionally, with a 1-cm tumor of this pathology subtype, her immunocompromised state is not an indication for Mohs micrographic surgery, either. The radiation was remote from the area she developed her basal cell carcinoma, so it is not an indication, either.
A 39-year-old woman is evaluated because of a 6-month history of a growth on the face. A biopsy is planned. Which of the following findings on pathology can be safely treated with observation only in this patient?
A) Cylindroma
B) Eccrine poroma
C) Nevus sebaceous
D) Trichoepithelioma
The correct response is Option D.
Trichoepitheliomas are neoplasms of follicular differentiation. Trichoepithelioma usually presents as multiple, yellowish-pink, translucent papules distributed symmetrically on the cheeks, eyelids, and nasolabial area. The lesions are more frequently seen in women. Lesions are benign but can be confused with basal cell carcinomas clinically and histologically. As they are benign, no further measures should be taken. However, in cases of desmoplastic trichoepithelioma, complete excision or Mohs surgery may be needed to clearly differentiate this entity from a carcinoma.
Eccrine poroma occurs as a solitary lesion usually on the sole of the foot or the palm of the hand in persons older than 40 years. It may also occur on the chest, the neck, or other locations. Eccrine poromas are seen as firm papules less than 2 cm in size. Lesions may occasionally be pedunculated and have a normal or erythematous color and a firm consistency. In rare instances, malignant eccrine poroma or porocarcinoma develops either spontaneously or from long-standing benign eccrine poroma. Treatment is surgical excision.
Verrucous nevus consists of closely set verrucous papules that may coalesce to form well-demarcated plaques. They may be skin colored, brown, or gray-brown. A linear configuration is common, especially for lesions on the limb. Such lesions may appear to follow skin tension lines. On histologic evaluation, there is hyperkeratosis, acanthosis, and papillomatosis. The histologic appearance is essentially that of a benign papilloma. Excision is the most reliable treatment. This may not be practical or advisable if the lesion is extensive or at sites not amenable to simple surgery. The excision should extend to the deep dermis; otherwise, the lesion may recur. Alternative treatments have included laser cryotherapy and electrodesiccation dermabrasion.
Cylindroma presents as either solitary or multiple lesions. Nodules may also be present on the face and rarely on the extremities. The lesion appears in adulthood. The surface is smooth and may be telangiectatic. Cylindromas are usually benign, but malignant changes have been reported. For solitary lesions, treatment is by excision or electrosurgery. For small cylindromas, the carbon dioxide laser may be used. Multiple cylindromas usually require extensive plastic surgery that may be obviated by progressively excising a group of nodules in multiple procedures.
Nevus sebaceous is a distinctive growth most commonly found on the scalp, followed by the forehead and retroauricular region. A nevus of epithelial and nonepithelial skin components, nevus sebaceous sustains age-related modifications in morphologic appearance. The nevus occurs singly and is asymptomatic. Two thirds are present at birth; the remaining third develop in infancy or early childhood. Male and female infants are equally affected. The three-stage evolution of the nevoid condition (newborn, puberty, and adult) parallels the natural histologic differentiation of normal sebaceous glands. In approximately 20% of patients, a third phase of evolution involves the development of secondary neoplasia in the mass of the nevus. A number of benign and malignant “nevoid tumors” may occur, the most common of which is the basal cell epithelioma. The malignant degenerations are relatively low grade; only a few instances of metastasis have been reported. Surgical excision of a nevus sebaceous is recommended because of the high potential for development of basal cell carcinoma and other tumors. The lesion should preferably be excised before puberty because it may enlarge, and the risk of malignant transformation increases after puberty.
A 55-year-old woman undergoes biopsy of a suspicious lesion on the dorsal hand. Which of the following is the most common malignant skin tumor of the hand?
A) Basal cell carcinoma
B) Keratoacanthoma
C) Melanoma
D) Merkel cell carcinoma
E) Squamous cell carcinoma
The correct response is Option E.
Malignant tumors are uncommon in the hand, and squamous cell cancer is by far the most common malignant tumor. Approximately 75% of malignant skin lesions on the hand are squamous cell cancer and appear as crusty, scaly raised lesions predominantly on the dorsal skin. They will occasionally ulcerate. Basal cell cancer is a common malignant skin tumor but only accounts for about 10% of hand skin cancers. Melanoma is unusual on the hand; it is responsible for about 3% of hand tumors. Merkel cell carcinoma of the hand is rarely found. Keratoacanthoma is a mimicker of squamous cell carcinoma with a much more rapid growth rate and spontaneous remission.
Sebaceous carcinoma most frequently affects which of the following anatomic sites?
(A) Eyelid
(B) Finger
(C) Nose
(D) Tongue
(E) Trunk
The correct response is Option A.
Sebaceous carcinoma arises from the adnexal epithelium of the sebaceous glands. It can be divided into two subtypes; the ocular subtype, which is most common, typically arises from the meibomian gland of the eyelid. This is a more aggressive carcinoma, frequently resulting in metastasis. In contrast, extraocular lesions are common to hair-bearing areas, and metastases are infrequent.
A 32-year-old woman is referred by her dermatologist for evaluation and treatment of a painful skin lesion on her lateral cheek. Examination shows a 5-mm raised skin lesion with a bluish hue. Excisional biopsy with 1-mm margins is performed. Pathologic analysis shows a well-circumscribed tumor arising within the dermis with diffuse dense basophilic cellular proliferation, eosinophilic hyaline deposits, and a lymphocytic infiltrate. The pathologic diagnosis is spiradenoma. Which of the following is the most appropriate next step in management of this lesion?
A) Chemotherapy targeting lymphocytes
B) Radiation therapy
C) Re-excision with 1-cm margins
D) Sentinel lymph node biopsy
E) Reassurance and observation
The correct response is Option E.
The most appropriate next step in management of a completely excised spiradenoma is reassurance and observation. Spiradenomas are well-differentiated, benign, dermal neoplasms. Their origin is controversial, with some believing that they originate from sweat glands and others believing that they originate from hair follicles. Most spiradenomas are seen in patients between 15 to 35 years of age. They usually present as small, solitary, painful nodules that can grow to several centimeters, often with a bluish hue. Spiradenomas usually arise on the head, neck, and trunk. There are various morphological subtypes, and they can occur alongside cylindromas, trichoepitheliomas, and/or trichoblastomas. Patients with Brooke-Spiegler syndrome have multiple spiradenomas, cylindromas, and trichoepitheliomas.
A 44-year-old woman has had a persistent nodular lesion measuring 5 mm in diameter on the anterior scalp for the past year. Pathologic evaluation of a specimen obtained on excisional biopsy shows dermatofibrosarcoma protuberans. Surgical resection with a 1-cm border and scalp rotation is used for closure. Which of the following is the most likely long-term outcome?
(A) Brain metastasis
(B) Local recurrence
(C) Pulmonary metastasis
(D) Regional nodal metastasis
The correct response is Option B.
Dermatofibrosarcoma protuberans (DFSP) is a low- to intermediate-grade sarcoma that typically is aggressive and has a local recurrence rate of up to 60%. Tumor cells tend to invade local tissue with tentacle-like projections in skin that appears normal clinically. The head and neck are involved in 14% of affected patients, and these tumors account for 1.4% of all head and neck sarcomas. In the head and neck region, the scalp and supraclavicular fossa are most often involved. Recurrence rates are higher for head and neck lesions (in up to 75% of affected patients) than rates in other sites, possibly because the extent of surgical resection is limited. Regional and distant metastases are uncommon, occurring in less than 4% of affected patients.
Treatment of DFSP includes wide surgical resection with margins of 3 cm. Many authors suggest that Mohs’ surgery is associated with a decreased rate of recurrence in DFSP. The effectiveness of adjuvant radiation has not been clearly substantiated.
Patients with erythroplasia of Queyrat have squamous cell carcinoma affecting which of the following sites?
(A) Arm
(B) Ear
(C) Nose
(D) Penis
(E) Scalp
The correct response is Option D.
Erythroplasia of Queyrat is an irregular, erythematous patch of full-blown squamous cell carcinoma or carcinoma in situ that affects the penis. It is a feature of the condition known as Bowen’s disease, which is characterized by widespread erythematous, brown, scaly patches. These lesions are composed of squamous cells that can completely replace normal dermis at the affected sites. Because approximately 15% of Bowen’s disease lesions progress to become invasive squamous cell carcinoma, surgical excision of an erythroplasia of Queyrat is recommended.
A 36-year-old woman, gravida 1, para 1, with a history of cesarean delivery is evaluated for a painless, firm, 6-cm wide subcutaneous mass that is fixed to the anterior abdominal wall with no associated symptoms. The mass was noted after pregnancy. Tumor markers are within normal limits. She has a family history of familial adenomatous polyposis (FAP) syndrome. Which of the following tumors is the most likely diagnosis?
A) Dermatofibrosarcoma protuberans
B) Desmoid
C) Lipoma
D) Lymphoma
E) Neurofibroma
The correct response is Option B.
Abdominal wall tumors are rare, accounting for less than 10% of all soft-tissue tumors. Desmoid tumors and soft-tissue sarcomas account for 45% and 40%, respectively, of all abdominal wall tumors. Desmoid tumors, also known as aggressive fibromatosis, are characterized by unpredictable progression or spontaneous regression, but lack the ability to metastasize. The majority of desmoid tumors arise from sporadic mutations in CTNNB1, whereas 10% arise in association with an APC mutation in familial adenomatous polyposis (FAP) syndrome. Abdominal wall desmoid tumors demonstrate an increased prevalence in women of childbearing age.
Dermatofibrosarcoma protuberans (DFSP) is an uncommon, locally aggressive abdominal wall tumor with low metastatic potential. DFSP originates from cutaneous tissues and is limited to superficial structures. The majority of abdominal wall DFSP occur in adults aged 20 to 50 years with similar sex distribution, and tumors are small (<5 cm) with characteristic purple or blue discoloration.
Neurofibromas appear as soft, skin-colored papules or small, subcutaneous nodules. The majority of neurofibromas are localized and arise from sporadic neurofibromin 1 gene (NF1). While the plexiform type is pathognomonic for hereditary neurofibromatoses caused by germline mutations in the neurofibromin 1 gene (NF1) or neurofibromin 2 gene (NF2).
Lipomas are common benign tumors composed of mature adipose cells. Lipomas usually develop on the trunk or proximal limbs as discrete rubbery masses in the subcutaneous tissues that present at any age. Among solitary cutaneous lipomas, 60% display clonal alterations, which are not associated with presentation of multiple lipomas. There is an increased prevalence of solitary lipomas in women, and multiple lipomas occur more frequently in men.
Primary abdominal wall lymphoma is a rare extra-nodal presentation with increased prevalence among male patients. Extra-nodal disease may present at all ages, but more that 75% of patients are over age 50 years. Patients with atypical lymphoma may not present with fever, night sweat, weight loss or anemia. Lactate dehydrogenase tumor marker is observed in extra-nodal lymphoma. Most soft-tissue lymphomas are of B-cell origin.
A 65-year-old man comes to the office with a 2-cm basal cell carcinoma involving the left nasal ala extending across the nasofacial junction onto the cheek. A photograph is shown. Medical history includes previous melanoma. Which of the following is an indication for Mohs micrographic surgery over conventional excision in this patient?
A) Cancer size
B) Diagnosis of basal cell carcinoma
C) History of previous melanoma
D) Location of the lesion
E) Patient age
The correct response is Option D.
Mohs micrographic surgical technique has demonstrated cure rates of 99% for primary basal cell carcinomas and up to 95% for recurrent basal cell carcinomas. In this particular patient, the strongest indication for use of the Mohs technique is the anatomical location. The nose is considered a high-risk location in the classically described “H-zone.” This patient underwent Mohs excision with multi-stage forehead flap reconstruction, as shown in the photographs. Patient age, history of previous melanoma, and tumor size 2 cm or less are not standard indications for Mohs excision. Other indications for Mohs technique include the following: Recurrent basal cell/squamous cell carcinomas, Locations prone to recurrence- “H-zone” of the face: periorbital, periauricular, temple, upper lip, nose/nasolabial fold, and chin, Tumors involving critical structures such as the eyelid or lip, Functionally important areas such as the genitals, perianal location, hands, and feet, Tumors arising in sites of previous irradiation therapy, Large tumors (> 2 cm) Lesions with ill-defined tumor margins, Histologic aggressive subtype (morpheaform, basosquamous, perineural, and invasive/poorly differentiated squamous cell carcinoma) Tumors arising in immunosuppressed patients such as transplant recipients or patients with genetic predisposition (basal cell nevus syndrome, xeroderma pigmentosum)
An 86-year-old woman comes to the office because of an 18-month history of the isolated, painful lesion shown. History includes CREST (calcinosis cutis, Raynaud phenomenon, esophageal dysfunction, sclerodactyly, and telangiectasia) syndrome. Which of the following is the most appropriate initial management?
A ) Debridement
B ) Hyperbaric oxygen therapy
C ) Injection of corticosteroids
D ) Measurement of serum calcium level
E ) Measurement of serum uric acid level
The correct response is Option A.
CREST syndrome is a member of the heterogeneous sclerodermas. Calcinosis cutis is the pathologic calcification of soft tissues and skin.
The calcific deposits, when symptomatic, can be tender and painful. They can ulcerate, drain a white chalky substance, and become secondarily infected. In scleroderma, calcific deposits are found predominantly in the extremities and around joints and bony prominences. Deposits are typically found in the flexor surfaces of the hands and the extensor surfaces of the forearms and knees. The deposits rest in the dermis but can also be found in deeper periarticular tissues.
Serum calcium, phosphorus, and alkaline phosphatase levels typically are normal; serum measurement does not contribute to management.
CREST is not associated with gout, so measurement of serum uric acid is noncontributory. Autografting prior to wound debridement and removal of calcific deposits is not recommended. Hyperbaric oxygen therapy may follow debridement.
No consistently reliable pharmacologic treatment, including injection of corticosteroids, has been shown to prevent or eliminate calcinosis. However, surgical excision of localized, painful large deposits can relieve symptoms; recurrence is rare. If calcinosis is diffuse, recurrence is more common. Successful palliation and significant remission of calcinosis using a carbon dioxide laser for debridement was shown in two case reports with a total of seven patients.
Calcinosis cutis is distinct from calciphylaxis, which is a poorly understood and highly morbid syndrome of vascular calcification and skin necrosis typically seen in 1 to 4% of patients with end-stage renal disease. Wound debridement in this setting is controversial and may cause exacerbation of skin necrosis.
A 17-year-old girl presents to the clinic to have the lesion in the photograph removed. She has had it since birth. She does not like its appearance and has noted it has changed in size and shape during the last two years. Histopathologic examination of the specimen is most likely to show which of the following?
A) Amelanotic melanoma
B) Basal cell carcinoma
C) Nevus sebaceous
D) Squamous cell carcinoma
E) Trichoblastoma
The correct response is Option C.
The photograph shows a sebaceous nevus of Jadassohn, and on pathology, a benign organoid nevus is the most likely finding. Given the patient’s age, malignant transformation is very rare (around 2.5%). Until recently, the most common transformation has been thought to be basal cell carcinoma (1.1%). However, it is now thought that many of these were trichoblastomas. A melanoma present since childhood does not fit the presentation of this patient.
Which of the following medications is most appropriate to prevent and to treat heterotopic ossification?
(A) Calcitonin
(B) Etidronate
(C) Mithramycin
(D) Phosphate
The correct response is Option B.
Heterotopic ossification is an abnormal proliferation of bone that occurs in patients who have sustained burns, spinal cord injuries, and closed head trauma. This condition becomes apparent initially on radiographs one to four months after injury. It may occur in up to 40% of patients who sustain spinal cord injuries and is clinically significant in one-half of those patients.
Etidronate (Didronel) and pamidronate (Aredia) are most appropriate for prevention or treatment of heterotopic ossification. These agents act primarily on bone to inhibit the formation, growth, and dissolution of hydroxyapatite crystals and do not impair fracture healing.
Calcitonin, mithramycin, and phosphate decrease serum calcium levels and do not directly affect heterotopic ossification.
A 50-year-old man has a keratoacanthoma on the left arm. He underwent removal of skin tumors on three previous occasions. His brother and father have had similar findings; the father subsequently died of a malignant tumor. Which of the following is the most appropriate diagnostic test?
(A) Bone scan
(B) Colonoscopy
(C) CT scan of the chest
(D) Panoramic radiograph
(E) Plain radiographs of the long bones
The correct response is Option B.
This patient has findings consistent with Muir-Torre syndrome, an autosomal dominant disorder that typically has its onset between the ages of 50 and 70 years. Multiple skin malignancies (eg, keratoacanthomas, basal and squamous cell carcinomas, adenocarcinomas) are characteristic. Because malignancies of the colon, kidneys, bladder, ovary, pancreas, and breast are also associated and may even be present prior to the appearance of skin lesions, diagnostic testing (such as colonoscopy) should be performed in any patient with multiple keratoacanthomas, especially in this patient who has a family history of similar findings. The other diagnostic tests listed above would not be effective in detecting malignancies associated with Muir-Torre syndrome.
A 51-year-old woman has had multiple lesions around the nostrils for the past two years. She says that the lesions first appeared similar to pimples and have enlarged. A photograph is shown above. The lesions have not resolved with intralesional injections of corticosteroids. Findings on nasal smear and tissue cultures are negative. Histologic examination of a biopsy specimen of a lesion shows noncaseating epithelioid granulomata.
Which of the following is the most likely diagnosis?
(A) Cutaneous sarcoidosis
(B) Keloids
(C) Neurofibromatosis
(D) Rhinosporidiosis
(E) Wegener’s granulomatosis
The correct response is Option A.
In this 51-year-woman who has the findings shown in the photograph, the most likely diagnosis is cutaneous sarcoidosis. This condition occurs in 10 Caucasians and 35 African Americans per 100,000 of population. Although sarcoidosis typically affects the lungs and intrathoracic lymph nodes, cutaneous symptoms, such as maculopapular eruptions, sarcoidal plaques, lupus pernio lesions, and subcutaneous and cutaneous nodules, occur in 25% of patients. Histologic examination of a biopsy specimen of affected skin shows noncaseating granulomata and aggregates of epithelioid cells and giant cells containing Schaumann (asteroid bodies) within the dermis.
Appropriate treatment of cutaneous sarcoidosis is intralesional administration of a corticosteroid or oral administration of hydroxychloroquine sulfate or methotrexate. Oral corticosteroids are reserved for patients with severe cutaneous disfigurement or symptomatic involvement of internal organs. Surgical options are limited because of the systemic nature of this condition.
In patients with keloids, histologic examination shows extensive fibrous tissue with scattered collagen fibers but without a definite architecture or a granulomatous pattern.
Neurofibromatosis is characterized histologically by swirls of fibroblastic cells, collagen, and nerve fibers. Granulomas are not present.
Rhinosporidiosis is a fungal granulomatous condition of the nose caused by Rhinosporidium seeberi bacteria. Affected patients have pedunculated masses with reddish-purple discoloration. Although findings on culture are usually negative, microscopic examination of nasal smears typically shows sporangia.
Wegener’s granulomatosis is a systemic disease that initially causes localized perforation of the septum or multiple intranasal ulcerations. Histologic examination shows findings consistent with necrotizing granulomatous vasculitis.
Which of the following peripheral nerve tumors is most commonly associated with von Recklinghausen disease?
A) Astrocytoma
B) Glioblastoma
C) Neurilemoma
D) Neurofibroma
E) Schwannoma
The correct response is Option D.
A neurofibroma is a lesion of the peripheral nervous system, which is derived from Schwann cells, other perineural cell lines, and fibroblasts. Neurofibromas may arise sporadically, or in association with von Recklinghausen disease (neurofibromatosis 1 or NF1). A neurofibroma may arise at any point along a peripheral nerve, and comes in two varieties.
The plexiform neurofibromas are larger tumors that develop inside the body and tend to intimately involve the nerves, blood vessels, and other structures in the body. They can reside deep inside the body or closer to the skin. Plexiform neurofibromas can cause pain, numbness, weakness, and disfigurement. These tumors do have a small chance of becoming cancerous. Plexiform neurofibromas may also be asymptomatic.
Dermal (subcutaneous) neurofibromas are small, nodule-like tumors that grow on or just under the surface of the skin. They can be painful, itchy, disfiguring, or tender when touched, but they have no known potential to become cancerous. Dermal neurofibromas may also be asymptomatic.
Schwannomas are peripheral nerve sheath tumors that can be seen with NF1, but are more commonly associated with neurofibromatosis 2. The major distinction between a schwannoma and a solitary neurofibroma is that a schwannoma can be resected while sparing the underlying nerve, whereas resection of a neurofibroma requires the sacrifice of the underlying nerve. A neurilemoma is another name for a schwannoma.
Astrocytomas and glioblastomas are tumors of the central nervous system. Astrocytomas and optic gliomas can be seen in association with NF1.
A 59-year-old man has a 1.5-cm lesion over the fourth extensor compartment. A morpheaform basal cell carcinoma was excised at the site of the lesion seven years ago. There is no extensor adherence or adenopathy. Which of the following is the most appropriate management of this lesion?
(A) Observation
(B) Marginal excision
(C) Mohs’ micrographic resection
(D) Wide local excision
(E) Radical resection
The correct response is Option C.
This patient has a recurrence of basal cell carcinoma, a common, slow growing, nonmelanotic skin tumor that often goes unnoticed and undiagnosed. Basal cell carcinoma is six to 18 times more common than squamous cell carcinoma; sun exposure is directly associated with the development of this type of tumor. Approximately 3% of all basal cell carcinomas are found on the upper extremities. There can be as many as 26 different histologic subtypes of basal cell carcinoma, including nodular, superficial, ulcerative, micronodular, infiltrative, sclerosing, morpheic, and mixed types. The infiltrative, ulcerative, sclerosing, and morpheaform types are the most aggressive.
Several treatment options for basal cell carcinoma are associated with similar cure rates. Lesions smaller than 6 mm can be managed with curettage and electrodesiccation; radiation therapy, cryotherapy, and topical application of 5-fluorouracil are also effective. For most basal cell carcinomas (excluding sclerosing and morpheaform variants), there is a 95% cure rate when the tumor is resected with a margin of 4 mm or smaller. In contrast, aggressive lesions and recurrent tumors are best treated with wide resection with margins greater than 5 mm or controlled resection using Mohs’ micrographic technique. However, Mohs’ surgery will render more exact margins and is thus preferred over wide resection alone, even if the wide margins are examined by frozen section. Following excision, the wound should be covered with a skin graft, which will prevent dessication of the underlying structures and allow for histologic examination of the lesion without further risk for local tumor spread.
Radical resection is not indicated for locally recurrent morpheaform basal cell carcinoma.
An otherwise healthy 65-year-old woman comes to the office because she noticed a well-circumscribed 2-cm dark brown lesion with a waxy surface on her back two years ago. On excision of the lesion, which of the following is the most likely pathologic diagnosis?
(A) Actinic keratosis
(B) Congenital nevus
(C) Cutaneous horn
(D) Keratoacanthoma
(E) Seborrheic keratosis
The correct response is Option E.
Seborrheic keratosis is commonly excised to prevent potential confusion with malignant melanoma. These lesions are often seen in large numbers on middle €‘aged and older patients. They are sharply circumscribed, waxy, and friable with a €œstuck on € appearance. Pigmentation ranges from tan to deep black. Treatment options include shave excision, electrodesiccation, freezing with liquid nitrogen, or simple excision. These lesions do not undergo malignant degeneration and are not easily confused with squamous cell carcinoma.
Actinic keratoses are premalignant lesions. These dry, scaly, rough patches range in color from yellow to dark brown and can progress to squamous cell carcinoma. A congenital nevus is seen in childhood and does not develop in later life. A cutaneous horn has significant keratotic growth and does not have a waxy appearance. Keratoacanthoses are rapidly growing lesions and do not have the appearance described.
Which of the following pigmented nevi is classified as a hamartoma?
(A) Acral nevus
(B) Becker nevus
(C) Dysplastic nevus
(D) Halo nevus
(E) Spitz nevus
The correct response is Option B.
A Becker nevus is a common benign cutaneous hamartoma with epidermal and/or dermal elements. These lesions can occur at birth but often develop during the second and third decades of life, and males are affected more frequently. Becker nevi are typically brown patches or plaques on the upper trunk. Hypertrichosis is commonly associated, and an underlying smooth muscle hamartoma is present. The term “nevus” is actually a misnomer because histologic examination of this lesion shows no nevus cells.
Acral nevi are typically junctional or compound and are located on the palmar or plantar surfaces.
A dysplastic nevus is referred to by several terms, including Clark’s nevus, atypical nevus, or atypical mole. This lesion is a precursor to malignant melanoma.
A halo nevus, or Sutton’s nevus, is a central melanocytic nevus surrounded by a rim or halo of hypopigmented skin. This loss of pigmentation typically corresponds with spontaneous regression of the central nevus and is thought to occur secondary to a cell-mediated and humoral immune response to nevus antigens.
A Spitz nevus is a benign proliferation of melanocytes on the face, trunk, or extremities. This lesion develops in children and is characterized by a rapid, irregular growth phase. Because it is difficult to differentiate from malignant melanoma, excision for histopathologic confirmation is recommended.
A 48-year-old man presents with an asymptomatic, slowly growing violaceous nodule overlying his right scapula. Dermoscopy shows a delicate pigmented network, vessels, and structureless light brown areas. Biopsy identifies the lesion as a dermatofibrosarcoma protuberans. To minimize recurrence, surgical excision should include which of the following?
A) Bone
B) 6-cm margin
C) Deep fascia
D) Frozen sections
E) Periosteum
The correct response is Option C.
Dermatofibrosarcoma protuberans (DFSP) is a rare neoplasm of intermediate malignancy. Taylor first described it in 1890, but Darier is credited with establishing DFSP as a distinct clinicopathological entity in 1924, and finally, Hoffman established the term in 1925.
Most DFSPs occur on the trunk (42%), followed by the upper extremities (23%), lower extremities (18%), then the head and neck (16%). The reason for recurrence is that microscopic projections are not removed adequately or assessed satisfactorily; the wider the margin, the higher the probability the tumor will be removed completely. Using 1-cm margins around the primary tumor leaves residual microscopic tumor in more than 70% of patients, 2-cm margins in 20 to 40% of patients, 3-cm margins in 9 to 15.5% of patients, and 5-cm margins in 5% of patients. Peripheral margins of 5 cm have a close to 0% recurrence rate. Wide resections may also not be practical in patients with tumors located in critical areas like the head and neck, and intraoperative frozen section assessments have not been reliable for determining margin status. Deep margins should always include excision of the deep fascia. The complete resection of the tumor requires excision of the external outer table in the cranium; muscle in trunk and extremities; peritoneum in thin patients with DFSP located on the abdomen, sternum, and clavicles; and vertebral apophysis when located on the thorax.
A 60-year-old man presents with a 6-mm lesion of the forehead. The patient states that it has enlarged over a period of 2 years. Examination of a biopsy specimen shows squamous cell carcinoma (adenoid subtype) with a 1.5-mm depth of involvement. Which of the following is the most likely risk factor for recurrence of this patient’s lesion after surgical excision?
A) Anatomic location
B) Depth of involvement
C) Growth rate
D) Histologic subtype
E) Size
The correct response is Option D.
A number of risk factors for recurrence have been identified for squamous cell carcinomas.
Histologic subtype is one such factor. Adenoid, adenosquamous, and desmoplastic subtypes are considered high risk for recurrence.
Anatomic location is another such factor and is typically considered in combination with the size of the lesion. High-risk areas include the “mask” areas of the face (eyelids, eyebrows, periorbital, nose, lips, chin, temple, ear), as well as genitalia, hands, and feet. Lesions greater than 6 mm in these areas indicate a high risk for recurrence. Mid-risk areas include the cheeks, forehead, scalp, and neck, with lesions greater than 10 mm indicating a high risk for recurrence. Low-risk areas include the trunk and extremities, with lesions greater than 20 mm indicating a high risk for recurrence. Based on this, this patient with a 6-mm forehead lesion would not be considered at high risk for recurrence.
Rapid growth is a risk factor for recurrence. However, this patient’s lesion grew gradually over a period of 2 years and not rapidly.
Depth of involvement greater than 2 mm indicates a high risk for recurrence.
Other risk factors for recurrence include poorly defined borders, immunosuppression, prior irradiation, site of inflammatory process, neurologic symptoms, moderate/poor differentiation, and perineural/vascular involvement.
For each patient with dermatologic symptoms, select the most likely diagnosis (A €“E).
A ) Bazex syndrome
B ) Erythroplasia of Queyrat
C ) Nevoid basal cell carcinoma syndrome
D ) Squamous cell cancer
E ) Xeroderma pigmentosum
A 5-year-old girl is brought to the office because of erythema, inflammation, and bullae formation on the face and neck area one day after spending a sunny day in the park with her family.
A 16-year-old boy comes to the office because of a three-year history of dome-shaped, tan papules on the face, neck, and trunk; erythematous pits in the palm of his hand; and swelling and pain in the molar and premolar areas.
The correct response for Item 12 is Option E and for Item 13 is Option C.
Xeroderma pigmentosum is an autosomally recessive disease with incidence of 1:1,000,000 in the United States and 1:40,000 in Japan (incidence in men and women is the same). The usual age of presentation is the first few years of life. Physical findings include acute sun sensitivity with sunburn-like reaction (erythema, inflammation, and bullae formation), pigmented macules, telangiectasias in photodistribution, dry, scaly, and atrophic skin, and multifold increased risk for BCC, squamous cell carcinoma, and malignant melanoma. Treatment includes sun avoidance, sunscreen, dermatologic cancer screening every three months, surgery, cryotherapy, topical treatment with 5-FU and imiquimod.
Nevoid basal cell carcinoma (Gorlin) syndrome is an autosomal dominant disease with a 1/60,000 incidence (incidence in men and women is the same). The usual age of presentation is birth or childhood. Physical findings include dome-shaped tan papules on face, neck, and trunk (basal cell carcinoma [BCC]); erythematous pits in the palm of the hand or plantar aspect of the feet; and swelling and pain in molar and premolar areas (odontogenic keratocysts), colobomas, hypertelorism, and fibrosarcomas. Differential diagnoses include Bazex syndrome, melanocytic nevi, and xeroderma pigmentosum. Diagnosis is usually made by clinical findings and supported by biopsy and skeletal surgery. Treatment includes surgical excision, application of topical 5-fluorouracil (5-FU) and imiquimod, and frequent cutaneous examinations. Patients should avoid radiotherapy, radiographs, and sun as these can induce the formation of new BCCs.
Bazex syndrome is another autosomally dominant inherited disorder. It is characterized by multiple BCCs of the face, follicular atrophoderma of the extremities, localized or generalized hypohidrosis, and hypotrichosis. Physical findings are classically unlike those of Gorlin syndrome and usually do not include pits in the hands and feet and molar pain.
Erythroplasia of Queyrat arises from the squamous epithelial cells of the glans penis. It is synonymous with Bowen disease of the glans penis, is seen mostly in uncircumcised men, and represents an in situ form of squamous cell carcinoma. Progression to invasive carcinoma may occur after a variable period of time.
A 12-year-old girl is brought to the office by her parents for consultation regarding a lesion on the posterior aspect of the scalp (shown). The lesion has been present since birth and has grown proportionately with the child. Which of the following is the most likely diagnosis?
(A) Compound melanocytic nevus
(B) Keratoacanthoma
(C) Nevus of Ito
(D) Nevus of Ota
(E) Nevus sebaceus of Jadassohn
The correct response is Option A.
The lesion shown represents a compound melanocytic nevus, which is a benign neoplasm composed mostly of melanocytes. Congenital melanocytic nevi, such as this one, are believed to represent an anomaly in embryogenesis and, as such, could be considered a malformation or a hamartoma. Melanocytic nevi are common in light- or fair-skinned patients and are relatively uncommon lesions in dark €‘skinned individuals. There is no sexual predilection, and melanocytic nevi are most common in children and young adults. Excisional biopsy is the treatment of choice.
Keratoacanthomas are round, firm, pinkish red or flesh-colored papules that rapidly progress to dome €‘shaped nodules with a central crateriform ulceration or a horn €‘like keratin plug. It is a relatively common low €‘grade malignancy that originates in the pilosebaceous glands and closely and pathologically resembles squamous cell carcinoma.
A nevus of Ota is a blue-gray lesion occurring on the face in the distribution of the trigeminal nerve (particularly the ophthalmic and maxillary branches €”V1 and V2).
A nevus of Ito is similar to the nevus of Ota but follows the distribution of the lateral brachial cutaneous and supraclavicular nerves.
A nevus sebaceus of Jadassohn is a yellow €‘orange, slightly elevated plaque that usually occurs on the face and scalp. It has the potential for malignant transformation to a basal cell carcinoma.
A 38-year-old African American man presents with multiple purulent tunneling lesions in bilateral axillae and his right groin. Which of the following surgical treatments will result in the lowest likelihood of recurrence?
A) Deroofing
B) Electrosurgical peeling
C) Incision and drainage
D) Skin-tissue-saving excision
E) Wide excision
The correct response is Option E.
Hidradenitis suppurativa (HS) is an inflammatory skin disease with a characteristic clinical presentation of recurrent or chronic painful or suppurating lesions in the apocrine gland-bearing regions. HS should be differentiated from infections such as furuncles, carbuncles, and abscesses (due to infectious agents and response to antibiotics), cutaneous Crohn disease (often concurrent with gastrointestinal Crohn, which has “knife-cut” ulcers and no comedones [whiteheads or blackheads]), and acne (distributed on the face and upper truncus, whereas HS predominantly affects intertriginous areas).
Surgery is required to definitively treat the tunnels and scars associated with chronic HS. Although surgery is commonly recommended, the literature supporting surgical treatment is anecdotal, composed mostly of large case series or retrospective study reports. A systematic review by Mehdizadeh et al. concluded that a lower recurrence rate was found in procedures with wide excision (overall, 13%; primary closure, 15%; using flaps, 8%; grafting, 6%) compared with local excision (22%) or deroofing (27%). These operations can be disfiguring, and despite the removal of significant amounts of tissue, do not necessarily protect against disease recurrence.
200A 17-year-old girl comes to the office because she has had lesions on the upper arms and in the axillary folds for the past eight years. Physical examination shows clusters of white, vesicular lesions ranging from 2 to 5 mm in diameter. The lesions are obliterated with gentle pressure but refill when pressure is removed. Diffuse swelling is palpable in the underlying subcutaneous tissue. Which of the following is the most likely diagnosis?
(A) Blue rubber bleb nevus syndrome
(B) Lymphangioma circumscriptum
(C) Maffucci syndrome
(D) Osler-Weber-Rendu syndrome
(E) Sebaceous nevus
The correct response is Option B.
Lymphangioma circumscriptum consists of clusters of small to moderate €‘sized, clear to whitish €‘appearing vesicular lesions. These lesions can be obliterated with gentle pressure, but they refill with the lymph when the pressure is removed due to the presence of communications with larger underlying cisterns of lymphatic malformation. Clear vesicles may be present on apparently normal skin or may top small papules. Although the classic lesion can occur anywhere on the body, it is particularly common over proximal parts of limbs and adjacent parts of the limb girdle including the upper arm, axillary, pectoral, and scapular regions. Adequate treatment of lymphangioma circumscriptum usually involves excision of the involved skin as well as the deeper lymphatic components.
Blue rubber bleb nevi usually present as raised blue to purple rubbery cutaneous lesions that are easily compressible. This is a rare morphogenetic disorder consisting of malformed vascular channels within the skin and bowel. When blood is expressed from the vesicles, the deflated blister becomes a wrinkled sac. When pressure is released, the deflated blisters refill with blood. The lesions sometime present at birth, but more frequently appear throughout adolescence. Subtotal excisions of tender or painful skin lesions can be of symptomatic relief.
Maffucci syndrome classically presents as blood €‘like, spongy, papular, or pedunculated vascular malformations in association with enchondromas. Visceral vascular malformations may also be present, and the involved bones are usually shortened and deformed. Approximately 20% of the enchondromas can degenerate into chondrosarcoma, which is usually manifest by increasing size and pain of a particular lesion.
Osler-Weber-Rendu syndrome presents as malformed ectatic vessels in the skin, mucous membranes, and viscera appearing after puberty and multiplying with advancing age. Hemorrhage from the lesions presents as epistaxis, hematemesis, hematuria, or melena.
Sebaceous nevi usually develop as firm, plaque-like, waxy-appearing yellowish lesions developing on the scalp. These are potentially premalignant lesions. Generally, these lesions should be excised, because there is an approximately 5% incidence of malignant degeneration, particularly during adolescence. Hair growth in these lesions is sparse to absent.
A 24-year-old woman is evaluated because of a slow-growing subcutaneous mass of the scalp. An excisional biopsy is performed, and pathologic examination shows keratin and its breakdown products. Which of the following is the most likely origin of this lesion?
A) Adipocyte
B) Capillary
C) Hair follicle
D) Mechanoreceptor
E) Sebaceous gland
The correct response is Option C.
Pilar cysts, also known as trichilemmal cysts, originate from the outer root sheath of the hair shaft. They present as firm, slow-growing subcutaneous nodules, and may be difficult to differentiate clinically from epidermoid cysts. They are commonly found on the scalp where they are the most common cutaneous cyst. They are lined by stratified squamous epithelium, which undergoes keratinization. In some cases, these lesions can demonstrate aggressive biologic behavior (proliferating trichilemmal tumors, malignant proliferating trichilemmal tumors), in which case they should be completely excised.
Lesions that originate from adipocytes include lipomas and angiolipomas.
Lesions of vascular origin include cherry angiomas and pyogenic granulomas.
A 53-year-old man presents with the lower eyelid skin lesions shown in the photograph. The lesions have been slowly growing over the past 12 months. Which of the following treatment options is the most appropriate?
A) Excision of the mass and overlying skin
B) Excision with 1-mm tissue margins
C) Excision with 2-mm tissue margins
D) Excision with 4-mm tissue margins and sentinel lymph node biopsy
E) Mohs micrographic surgery
The correct response is Option A.
The patient presents with xanthelasmata, which are localized accumulation of lipid deposits on the eyelids. Multiple treatment modalities are available, including chemical peels, cryotherapy, and laser ablation. Traditionally, surgical excision has been used and yields excellent cosmetic outcomes. As the lesions are benign, there is no indication for Mohs micrographic surgery and no margins are required.
A 58-year-old man comes to the office because of a 3-month history of multiple light-red, scaly lesions of the scalp and forehead. Physical examination shows extensive sun damage to the face and scalp, including multiple flat lesions measuring between 3 and 10 mm in diameter. Examination of a specimen obtained on shave biopsy shows actinic keratosis without invasive malignancy. In addition to daily application of sunscreen and wearing protective garments, which of the following is the most appropriate management?
A) Application of topical 5-fluorouracil
B) Excision of the lesions with 2-mm margins
C) Repeat biopsy in 6 months
D) Shave excision of the lesions
E) Observation only
The correct response is Option A.
Actinic keratosis is a common, premalignant lesion that is a direct result of sun damage. These lesions typically occur in fair-skinned patients who have an extensive history of solar injury. Lesions are flat or slightly raised, red, and scaly. They can be isolated or diffuse. Over time, they can progress to squamous cell carcinoma or other precancer lesions, such as Bowen disease (in situ squamous cell), cutaneous horns, and keratoacanthomas.
All patients must be counseled on sun protection measures, such as daily sunscreen application and wearing protective garments. Observation alone may be applicable for very small lesions or in certain infirm or elderly patients, but it is not appropriate in most circumstances because of the potential for malignancy. The rate of transformation is not clear but has been reported to be 1 to 25% per year per lesion.
Ideal treatment involves topical destructive measures, such as application of 5-fluorouracil cream. Typically, it is applied twice daily for 2 to 4 weeks. Other common treatments include imiquimod cream (Aldara), cryotherapy with liquid nitrogen, and photodynamic therapy with 5-aminolevulinic acid (Levulan). These topical treatments are preferable to excision of the superficial lesions described because they can typically treat multiple and diffuse lesions without extensive scarring. Local redness and irritation does occur and resolves over a period of weeks. Subclinical lesions can also be treated, preventing growth of new lesions. Excision or repeat biopsy should be reserved for isolated lesions that are refractory to less invasive measures. Chemical peels and laser resurfacing have also been described as treatment options.
An 85-year-old man who takes an anticoagulant medication comes to the office for evaluation of a recurrent 1-cm nodular basal cell carcinoma at his nasal tip that has started to bleed intermittently. Medical history includes placement of a cardiac stent 1 month ago, after myocardial infarction. Which of the following is the most appropriate treatment for this patient?
A) Electrodessication
B) Excision with forehead flap
C) Pembrolizumab therapy
D) Superficial radiation therapy
E) Topical application of 5-fluorouracil
The correct response is Option D.
With a 5-year recurrence rate of about 3% for nodular basal cell carcinomas (BCC), superficial radiation therapy has become a viable alternative to Mohs micrographic surgery, which remains the gold standard for treatment of nonmelanomatous skin cancers.
Electrodessication has a high recurrence rate and would subject this anti-coagulated patient to the risk of bleeding after surgery or thrombosis should his anticoagulation be discontinued.
Excision with forehead flap would not be safe for a patient with a recent myocardial infarction.
Topical 5-fluorouracil is not indicated in the management of nodular BCC.
Pembrolizumab therapy is indicated for Stage IV melanoma.
A 4-year-old boy is brought to the office because of a lump near his right eyebrow. The lesion has been enlarging gradually since it was first noticed by his parents when he was 3 months of age. On physical examination, a 2-cm, firm, mobile mass is palpated at the lateral aspect of the right eyebrow. The lesion has distinct margins and there are no other related symptoms. Which of the following is the most appropriate initial step in management?
A ) Angiography
B ) CT
C ) Needle aspiration
D ) Surgical excision
E ) Observation only
The correct response is Option D.
The most likely diagnosis in the patient described is a dermoid cyst of the orbital region. Most orbital dermoids manifest as solitary masses with distinct palpable margins and without intracranial extension. These lesions are present in the lateral aspect of the upper orbit and usually do not have any related symptomatology. As a result, these patients do not require preoperative evaluation to exclude the possibility of intracranial involvement. Simple excision of the cyst is appropriate at the time of diagnosis.
Rarely, a dermoid cyst may present with indistinct margins. During palpation, the lesion may appear to decrease in size and proptosis may occur. These rare patients may have extension of the dermoid cyst through the lateral orbital wall. In these cases, further evaluation with CT should occur to determine the extent of intraorbital involvement and determine whether more complex surgery is needed to reconstruct the defect. Failure to do this may result in an incomplete resection of the dermoid, possibly leading to inflammation, abscess, or sinus formation.
Observation is not recommended for management of dermoid cysts. These cysts should be appropriately managed with either surgical excision or further preoperative evaluation at the time of diagnosis. Needle aspiration has not been shown to be effective for preoperative evaluation of a dermoid cyst, nor has angiography. Dermoid cysts can occur bilaterally.
In contrast, midline dermoid cysts are much more likely to demonstrate intracranial involvement, and, therefore, immediate surgical resection is not recommended. A CT of the head, including the base of the skull, is needed to rule out intracranial extension prior to removal of the lesion and to rule out other types of lesions such as glioma, encephalocele, angiofibroma, and others. Splaying of the nasal bones on CT can be seen without intracranial extension. However, patients with intracranial involvement will also have defects in the foramen cecum and crista galli and significant dural attachment. These findings indicate the need for craniotomy for adequate exposure and resection of the lesion.
A 56-year-old man has a 30-degree flexion contracture of the proximal interphalangeal (PIP) joint of the right ring finger. He has had thickening in the palm at the base of the finger for the past five years. Which of the following structures are most likely involved in the PIP joint contracture?
(A) Central and spiral cords
(B) Lateral cord and knuckle pad
(C) Natatory and retrovascular cords
(D) Retrovascular and lateral cords
(E) Spiral cord and Cleland’s ligament
The correct response is Option A.
The central, lateral, and spiral cords each contribute to recurrent contracture of the PIP joint; the little finger is affected most frequently. The central cord develops from fascia between the neurovascular bundles and is continuous proximally with the pretendinous cord. It attaches distally to the tendon sheath over the middle phalanx. The lateral cord is adherent to the skin, while the spiral cord can occur as a continuation of the pretendinous cord or can arise at the musculotendinous junction of the intrinsic muscle. This cord straightens and courses less obliquely over time.
The natatory cord passes across the palm at the level of the web spaces and attaches to each individual flexor tendon sheath. Contracture of this cord can contribute to contracture of the PIP joint. Cleland’s ligaments are fascial structures located dorsal to the neurovascular bundle that help to hold the skin in position during flexion and extension of the finger. These structures are only an occasional cause of PIP joint contracture. The retrovascular cord most frequently causes contractures of the distal interphalangeal joint. This longitudinally oriented fascial cord lies dorsal to the neurovascular bundle and palmar to Cleland’s ligament.
Knuckle pads are a manifestation of joint contracture and not a cause themselves.
Which of the following lesions is most likely to contain malignant cells?
(A) Acrochordons
(B) Cutaneous horn
(C) Dermatofibroma
(D) Molluscum contagiosum
(E) Seborrheic keratosis
The correct response is Option B.
Cutaneous horns are most likely to contain malignant cells. These hard, cone-shaped cutaneous projections are typically caused by excessive epidermal growth and retention of keratin. Although cutaneous horns are most frequently associated with benign irregularities, approximately 20% are associated with premalignant lesions, and nearly 15% are associated with squamous cell carcinoma. Therefore, in patients with cutaneous horns, shave biopsy should be performed initially to exclude any underlying malignancy.
Acrochordons are simple skin tags not linked to malignancy. Dermatofibromas are benign, localized, fibrous tumors contained within the dermis that can develop following trauma. Molluscum contagiosum are virally induced papules commonly seen in children and young adults and may be associated with sexual transmission. Seborrheic keratoses are benign keratinocytic tumors seen in patients older than 30 years of age. Although these lesions often affect sun-damaged areas, they do not contain malignant cells.
A 32-year-old Caucasian woman presents with multiple (>50) brown lesions on her arms and lower legs. They appear to be in areas of sun exposure. On examination, many of these lesions are well circumscribed, even in color, and less than 5 mm in size. The patient has a family history of melanoma. There are too many lesions to excise. Which of the following findings in one of these lesions would prompt an excisional biopsy?
A) Asymmetry
B) Clearly demarcated borders
C) Dark coloration
D) Waxy surface
The correct response is Option A.
In this patient with multiple melanocytic nevi, lesions should be treated with excisional biopsy if there is a high suspicion for melanoma. As there are more than 50 lesions, clearly there are too many to excise. These lesions should be evaluated for asymmetry, border irregularity, variable color, diameter greater than 6 mm, and evolution. Any of these signs in a lesion should lead to an excisional biopsy with a suspicion of melanoma, especially given the patient’s family history.
Lesions with a waxy surface are seborrheic keratoses and commonly found in an elderly population in sun-exposed areas. Dark coloration does not lead to a suspicion of melanoma.
A 46-year-old woman has had painful purpura-like lesions on the breasts, abdomen, and lower extremities for the past three weeks. She currently undergoes dialysis for end-stage renal disease. Physical examination shows several dry, leathery, full-thickness areas of skin necrosis. A photograph is shown above. Calciphylaxis is diagnosed.
Laboratory studies are most likely to show an increased serum level of which of the following?
(A) Calcium
(B) Glucose
(C) Parathyroid hormone
(D) Protein C
(E) Thyroid-stimulating hormone
The correct response is Option C.
Calciphylaxis, also known as uremic gangrene syndrome, is a rare complication of end-stage renal disease. This condition is characterized by painful, ischemic, violaceous skin lesions on the extremities and sometimes the trunk that exhibit a livido reticularis pattern. Full-thickness skin necrosis ensues, leading first to secondary infection, then to sepsis, and frequently to death; the mortality rate in patients with calciphylaxis has been reported to be as high as 60%. Histologic examination of affected skin shows fat necrosis, calcification of the subcutaneous tissues, and microcalcifications in the intima and media of small arteries. Luminal narrowing of the vessels with intimal hyperplasia is common, and complete occlusion also occurs.
Calciphylaxis is a result of the metabolic imbalance in calcium and phosphate homeostasis that is frequently observed in patients with renal failure. Secondary hyperparathyroidism causes an abnormal increase in calcium X phosphate product, which can lead to soft-tissue calcification. In these patients, serum levels of parathyroid hormone are most likely to be increased. Hyperphosphatemia is also characteristic.
The lesions that occur in patients with calciphylaxis rarely heal spontaneously and frequently become infected. Therefore, various management options should be considered, including debridement of the lesions, topical administration of antimicrobial agents, and frequent dressing changes, followed by skin grafting, or direct excision and closure of the lesions. However, in this patient, the dry, leathery composition of the affected skin suggests that infection has not yet developed. Subtotal thyroidectomy is also advocated for treatment in some patients.
Although hypercalcemia may be present, serum calcium levels are normal in most patients with this condition.
Many patients with end-stage renal disease also have diabetes mellitus, but hyperglycemia is not typically associated with calciphylaxis.
A deficiency of, and not an increase in, serum protein C levels has been proposed to be a contributing factor to tissue death in these patients because it causes thrombosis in small vessels.
Increased serum levels of thyroid-stimulating hormone occur in patients with hypothyroidism.
A 50-year-old woman with a history of scleroderma is evaluated because of a 1.5-cm lesion on her right cheek. Patient history includes basal cell carcinoma excision at the same site 3 years ago. A punch biopsy shows basal cell carcinoma (micronodular subtype). Which of the following is the most appropriate indication for Mohs micrographic surgery in this patient?
A) Histologic subtype
B) History of scleroderma
C) Location of lesion
D) Recurrence of lesion
E) Size of lesion
The correct response is Option D.
Mohs micrographic surgery is a surgical technique in which tumor excision and microscopic examination of tissue margins are performed by the same surgeon. Use of a beveled excision and careful mapping of the peripheral and deep margins of horizontal frozen sections allow for comprehensive examination of all the borders of the excised tissue, resulting in excellent cure rates. In addition to the high cure rate, Mohs surgery is a tissue-sparing procedure that is an important advantage in cosmetically and functionally sensitive areas and contrasts with traditional approaches in which a set margin of excision is performed.
Indications for Mohs surgery include recurrent basal cell carcinomas (BCC) and squamous cell carcinomas (SCC), locations prone to recurrence (“H-zone” of the face: inner canthus, nasolabial fold, nose, periorbital, temple, upper lip and periauricular regions, retroauricular, and chin), at/near critical structures (e.g., eye, lip), large tumors (>2 cm), ill-defined tumor margins, aggressive histology (BCC - morpheaform infiltrative, basosquamous, perineural; SCC - poorly differentiated, invasive, perineural), and special hosts (immunosuppressed, basal cell nevus syndrome, xeroderma pigmentosum). Therefore, in this patient, the primary indication for Mohs surgery would be the recurrent nature of her BCC.
An otherwise healthy 30-year-old woman is diagnosed with dermatofibrosarcoma protuberans (DFSP) of the upper back. A wide excision is performed, and a local flap is used to reconstruct her back. On follow-up evaluation, CT scan shows multiple pulmonary metastases. Which of the following is the most appropriate next step in management?
A) Chemotherapy
B) Hormone therapy
C) Immunotherapy
D) Radiation therapy
E) Surgical excision
The correct response is Option A.
The most appropriate next step in management is chemotherapy. Patients with inoperable, recurrent, or metastatic disease may benefit from imatinib which is a tyrosine kinase inhibitor and acts as a molecularly targeted drug. It acts by inhibiting the platelet-derived growth factor receptor tyrosine kinase. Dermatofibrosarcoma protuberans (DFSP) is characterized by chromosomal rearrangements resulting in the production of platelet-derived growth factor B, eventually leading to autocrine growth stimulation of DFSP. Imatinib functions as an inhibitor of platelet-derived growth factor receptors, thus blocking this autocrine stimulation. Therefore, imatinib can be used as an adjuvant therapy for cases in which obtaining sufficient surgical margins is impossible. Neoadjuvant imatinib has also been used for locally advanced primary tumors.
Radiation therapy may improve local control and reduce the risk of recurrence postoperatively in patients with DFSP. There is no described role of it in pulmonary metastases. Resection of the multiple lesions in the lung, and hormonal or immune therapy are not recommended for metastatic DFSP.
A 30-year-old woman with melasma comes to the office for consultation regarding treatment options. Examination shows Fitzpatrick type V skin. Which of the following treatments is most appropriate in this patient?
(A) Erbium laser
(B) Phenol/croton oil
(C) 1% Tretinoin
(D) 35% Trichloroacetic acid
The correct response is Option C.
Melasma may be difficult to treat in dark-skinned (Fitzpatrick type IV or V) patients due to side effects, most commonly hypopigmentation. For years, the gold standard for treating dark-skinned patients with melasma was alpha-hydroxy acid peel, such as with 70% glycolic acid. Recent data have shown a 1% tretinoin peel, weekly or biweekly, to be equally as effective as 70% glycolic acid for treating melasma in these patients.
Erbium laser treatment is not appropriate because it may cause hypopigmentation in dark-skinned patients. Neither phenol oil nor croton oil is indicated in dark-skinned patients because they may produce hypopigmentation. Also, 35% trichloroacetic acid (TCA) is not recommended in dark-skinned patients for similar reasons. However, 10% or 15% TCA may be used with fewer adverse effects in these patients.